Mathcenter Forum

Mathcenter Forum (https://www.mathcenter.net/forum/index.php)
-   ปัญหาคณิตศาสตร์ทั่วไป (https://www.mathcenter.net/forum/forumdisplay.php?f=1)
-   -   โจทย์ปัญหาคณิตศาสตร์ สวัสดีปีใหม่ 2548 ครับ (https://www.mathcenter.net/forum/showthread.php?t=724)

nooonuii 01 มกราคม 2005 23:02

7. จงพิสูจน์โดยไม่ใช้ Fermat's Last Theorem ว่า ไม่มีจำนวนเฉพาะ p,q,r ซึ่งสอดคล้องสมการ pn + qn = rn ทุกค่า n2

8. ให้ x,y,z เป็นจำนวนจริงบวก ซึ่ง xyz = 1 จงพิสูจน์ว่า
\[ (\frac{x^{2548}}{1+x+xy} + \frac{y^{2548}}{1+y+yz} + \frac{z^{2548}}{1+z+zx}) (\frac{y^{2548}}{1+x+xy} + \frac{z^{2548}}{1+y+yz} + \frac{x^{2548}}{1+z+zx}) (\frac{z^{2548}}{1+x+xy} + \frac{x^{2548}}{1+y+yz} + \frac{y^{2548}}{1+z+zx}) \geq 1 \]

gon 02 มกราคม 2005 00:29

ข้อ 2. นี่หาที่ผิดไม่เจอจริง ๆ nooonuii ช่วยชี้แนะด้วยครับ.

R-Tummykung de Lamar 02 มกราคม 2005 02:31

ข้อ 3 ตอบว่า [0,2) ถูกไหมครับ

R-Tummykung de Lamar 02 มกราคม 2005 10:11

ปัญหาหนอนแทะครับ
F I V E O U R N
1 2 5 4 9 8 0 3
1 2 5 6 9 8 0 3
1 2 5 7 9 8 0 3
1 2 6 4 9 7 0 3
1 2 6 5 9 7 0 3
1 2 6 8 9 7 0 3
1 2 7 4 9 6 0 3
1 2 7 5 9 6 0 3
1 2 7 8 9 6 0 3
1 2 8 4 9 5 0 3
1 2 8 6 9 5 0 3
1 2 8 7 9 5 0 3
1 4 5 2 9 8 0 3
1 4 5 6 9 8 0 3
1 4 5 7 9 8 0 3
1 4 6 2 9 7 0 3
1 4 6 5 9 7 0 3
1 4 6 8 9 7 0 3
1 4 7 2 9 6 0 3
1 4 7 5 9 6 0 3
1 4 7 8 9 6 0 3
1 4 8 2 9 5 0 3
1 4 8 6 9 5 0 3
1 4 8 7 9 5 0 3
1 5 6 2 9 7 0 3
1 5 6 4 9 7 0 3
1 5 6 8 9 7 0 3
1 5 7 2 9 6 0 3
1 5 7 4 9 6 0 3
1 5 7 8 9 6 0 6
1 6 5 2 9 8 0 3
1 6 5 4 9 8 0 3
1 6 5 7 9 8 0 3
1 6 8 2 9 5 0 3
1 6 8 4 9 5 0 3
1 6 8 7 9 5 0 3
1 7 5 2 9 8 0 3
1 7 5 4 9 8 0 3
1 7 5 6 9 8 0 3
1 7 8 2 9 5 0 3
1 7 8 4 9 5 0 3
1 7 8 6 9 5 0 3
1 8 6 2 9 7 0 3
1 8 6 4 9 7 0 3
1 8 6 5 9 7 0 3
1 8 7 2 9 6 0 3
1 8 7 4 9 6 0 3
1 8 7 5 9 6 0 3
2 1 7 3 9 8 0 5
2 1 7 4 9 8 0 5
2 1 7 6 9 8 0 5
2 1 8 3 9 7 0 5
2 1 8 4 9 7 0 5
2 1 8 6 9 7 0 5
2 3 7 1 9 8 0 5
2 3 7 4 9 8 0 5
2 3 7 6 9 8 0 5
2 3 8 1 9 7 0 5
2 3 8 4 9 7 0 5
2 3 8 6 9 7 0 5
2 4 7 1 9 8 0 5
2 4 7 3 9 8 0 5
2 4 7 6 9 8 0 5
2 4 8 1 9 7 0 5
2 4 8 3 9 7 0 5
2 4 8 6 9 7 0 5
2 6 7 1 9 8 0 5
2 6 7 3 9 8 0 5
2 6 7 4 9 8 0 5
2 6 8 1 9 7 0 5
2 6 8 3 9 7 0 5
2 6 8 4 9 7 0 5


แฮ่กๆๆ เหนื่อยเลยครับ :D
ใช้ computer serch(เลียนแบบคุณ warut) ครับ :D

aaaa 02 มกราคม 2005 10:12

เฉลยข้อ 8
สังเกตุว่า
\[1=\frac{1}{1+x+xy}+\frac{1}{1+y+yz}+\frac{1}{1+z+zx}\]
ใช้ Holder's inequality ได้คำตอบตามต้องการ
P.S. ข้อนี้ไม่จำเป็นต้องยกกำลัง 2548 จะเป็นกำลัง n ใดๅก็ได้

nooonuii 02 มกราคม 2005 12:09

ขอโทษครับพี่กร ทุกอย่างสมบูรณ์แบบแล้วครับ เป็นความผิดของผมเองครับ เบลอไปหน่อย :D

nooonuii 02 มกราคม 2005 12:48

9. จงหาพื้นที่ของรูปสี่เหลี่ยมนูน (convex quadrilateral) ซึ่งมีความยาวด้านเป็น 1,3,4,5 หน่วย และมีเส้นทแยงมุมเส้นหนึ่งยาว 5 หน่วย

10. จงพิสูจน์ว่า ทุกปีจะต้องมีวันศุกร์ที่ 13 อย่างน้อยหนึ่งวัน

11. จงหาจำนวนจุดที่น้อยที่สุดซึ่งเมื่อบรรจุในวงกลมหนึ่งหน่วย(รวมขอบวงกลม) แล้วจะต้องมีอย่างน้อยสองจุดที่ห่างกันไม่เกินหนึ่งหน่วย

12. จงพิสูจน์ว่าสมการ x!y! = z! มีคำตอบที่เป็นจำนวนเต็มบวกทั้งหมดเป็นจำนวนอนันต์ เมื่อ x>5

nooonuii 02 มกราคม 2005 13:07

13. ให้ a,b,c เป็นจำนวนจริงบวก จงพิสูจน์ว่า

1) (IMO'1995) ถ้า abc = 1 แล้ว 1 / a3(b + c) + 1/b3(c + a) + 1/c3(a + b) 3/2

2) (nooonuii) ถ้า abc = 1 แล้ว 1 / a2(b + c) + 1/b2(c + a) + 1/c2(a + b) 3/2

3) (nooonuii) ถ้า a + b + c = 3 แล้ว 1 / a(b + c) + 1/b(c + a) + 1/c(a + b) 3/2

aaaa 02 มกราคม 2005 13:09

ข้อ 12 นี่ผิดพลาดรึเปล่าครับ ถ้า x=0 มันเห็นชัดว่า y=z เป็นจำนวนเต็มไม่ติดลบใดๆก็ได้

aaaa 02 มกราคม 2005 13:20

เฉลยข้อ 11
ตอบ 6 จุด โดยการแบ่งวงกลมหนึ่งหน่วย ออกเป็น 6 ส่วนเท่าๆกันด้วยเส้นตรงหกเส้นที่ผ่านจุดศก
ห่างกันทีละ 60 องศา และประยุกต์ pigeonholes principle

aaaa 02 มกราคม 2005 13:25

เฉลย ข้อ 9
ตอบ \( 6+\frac{3\sqrt{11}}{4}\)

aaaa 02 มกราคม 2005 16:03

เฉลยข้อ 13
(i) โดยอสมการ Cauchy-Schwarz ได้ว่า
\[\left(\frac{1}{a^3(b+c)}+\frac{1}{b^3(c+a)}+\frac{1}{c^3(a+b)}\right)\left(a(b+c)+b(c+a)+c(a+b)\right)
\geq\left(\frac{1}{a}+\frac{1}{b}+\frac{1}{c}\right)^2=(ab+bc+ca)^2\]
ดังนั้น \(\text{LHS}\geq\frac{ab+bc+ca}{2}\) ใช้อสมการ AM-GM ได้ผลที่ต้องการ

(ii) พิจารณาเทอม
\[\frac{1}{a^2(b+c)}=\frac{abc}{a^2(b+c)}=\frac{bc}{ca+ab}\]
ทำนองเดียวกันได้ว่า
\[\frac{1}{b^2(c+a)}=\frac{ca}{ab+bc},\quad\frac{1}{c^2(a+b)}=\frac{ab}{bc+ca}\]
ให้ \(x=bc,y=ca,z=ab\) ได้อสมการโจทย์สมมูลกับ \(xyz=1\) และ
\[\frac{x}{y+z}+\frac{y}{z+x}+\frac{z}{x+y}\geq\frac{3}{2}
\]
ซึ่งพิสูจน์ได้ทำนองเดียวกับข้อ 13(i)

(iii) โดยอสมการ Holder ได้ว่า
\[\left(\frac{1}{a(b+c)}+\frac{1}{b(c+a)}+\frac{1}{c(a+b)}\right)\left(a+b+c\right)\left((b+c)+(c+a)+(a+b)\right)\geq
(1+1+1)^3
\]
ดังนั้น
\[\frac{1}{a(b+c)}+\frac{1}{b(c+a)}+\frac{1}{c(a+b)}\geq\frac{3}{2}
\]

nooonuii 02 มกราคม 2005 22:40

แก้ข้อ 12 ให้แล้วครับคุณ aaaa
ส่วนข้อ 11 ยังทำให้น้อยกว่านั้นได้ (มั้ง) ครับ :D ผมก็ไม่แน่ใจเหมือนกันว่าคำตอบผมถูกรึเปล่า แต่ผมลดลงมาได้ถึง 5 ครับ

ข้อ 9 คำตอบน่าจะเป็น 6+3/2 11 นะครับ
ข้อ 3 ของน้อง R-Tummykung de Lamar ถูกแล้วครับ

aaaa 03 มกราคม 2005 00:04

ข้อ 9 ผม check แล้วครับ พื้นที่สามเหลี่ยมส่วนที่มีด้านยาว 1,5,5 ส่วนสูงเท่ากับ \(\sqrt{25-\frac{1}{4}}=\frac{3\sqrt{11}}{2}\)
ดังนั้นพื้นที่เท่ากับ \(\frac{1}{2}\times1\times\frac{3\sqrt{11}}{2}=\frac{3\sqrt{11}}{4}\) ของผมถูกแล้วครับ

ข้อ 11 ถ้า 5 จุดผมทำให้แต่ละจุดห่างกันมากกว่า 1 หน่วยได้ครับ ลองนึกถึงรูปหกเหลี่ยมยาวด้านละ 1 หน่วย
ที่แนบในวงกลมหนึ่งหน่วย เลือกจุด 5 จุดให้ห่างกันมากกว่า 1 หน่วยได้ครับ

ข้อ 12 ถ้า x=1 ยังคงเลือกให้ \(y=z>0\) ใดๆก็ได้

aaaa 03 มกราคม 2005 00:12

ผมมีโจทย์มาถามครับ (Putnam Exam 1966)
ข้อ 1 จงหา
\[
\lim_{n\to\infty}\sqrt{1+2\sqrt{1+3\sqrt{1+\sqrt{\cdots+(n-1)\sqrt{1+n}}}}}
\]


เวลาที่แสดงทั้งหมด เป็นเวลาที่ประเทศไทย (GMT +7) ขณะนี้เป็นเวลา 15:24

Powered by vBulletin® Copyright ©2000 - 2024, Jelsoft Enterprises Ltd.
Modified by Jetsada Karnpracha